Phase 3 Test 2 Flashcards

1
Q
  1. The right atrium, right ventricle, and part of the left ventricle are supplied by the: A) circumflex artery.
    B) left anterior descending artery.
    C) left main coronary artery.
    D) right coronary artery.
A

D

How well did you know this?
1
Not at all
2
3
4
5
Perfectly
2
Q
  1. In contrast to the right side of the heart, the left side of the heart:
    A) drives blood out of the heart against the relatively high resistance of the systemic circulation. B) is a high-pressure pump that sends blood through the pulmonary circulation and to the lungs. C) is a relatively low-pressure pump that must stretch its walls in order to force blood through the aorta.
    D) drives blood out of the heart against the relatively low resistance of the pulmonary circulation.
A

A

How well did you know this?
1
Not at all
2
3
4
5
Perfectly
3
Q
  1. The amount of blood that is pumped out by either ventricle per minute is called: A) ejection fraction.
    B) cardiac output.
    C) stroke volume.
    D) minute volume.
A

B

How well did you know this?
1
Not at all
2
3
4
5
Perfectly
4
Q
  1. Cardiac output is influenced by: A) heart rate.
    B) stroke volume.
    C) heart rate and/or stroke volume. D) ejection fraction and heart rate.
A

C

How well did you know this?
1
Not at all
2
3
4
5
Perfectly
5
Q
  1. The presence of dizziness in a patient with a suspected myocardial infarction is MOST likely the result of:
    A) fear and anxiety.
    B) the effects of nitroglycerin.
    C) acute left-sided heart failure. D) a reduction in cardiac output.
A

D

How well did you know this?
1
Not at all
2
3
4
5
Perfectly
6
Q

Changes in cardiac contractility may be induced by medications that have a positive or negative ___________ effect. A) vasoactive
B) dromotropic
C) inotropic
D) chronotropic

A

C

How well did you know this?
1
Not at all
2
3
4
5
Perfectly
7
Q
  1. Stimulation of alpha and beta receptors affects the: A) heart only.
    B) heart and blood vessels.
    C) blood vessels and lungs.
    D) heart, lungs, and blood vessels.
A

D

How well did you know this?
1
Not at all
2
3
4
5
Perfectly
8
Q
  1. Vasoconstriction occurs following stimulation of: A) beta-1 receptors.
    B) beta-2 receptors.
    C) alpha receptors.
    D) alpha and beta receptors.
A

C

How well did you know this?
1
Not at all
2
3
4
5
Perfectly
9
Q
  1. Drugs that have alpha or beta sympathetic properties are called: A) vagolytics.
    B) sympathomimetics.
    C) parasympatholytics.
    D) adrenergic blockers.
A

B

How well did you know this?
1
Not at all
2
3
4
5
Perfectly
10
Q
  1. To increase myocardial contractility and heart rate and to relax the bronchial smooth muscle, you must give a drug that:
    A) stimulates beta-1 and beta-2 receptors.
    B) stimulates beta-2 and alpha receptors.
    C) blocks beta-1 and beta-2 receptors.
    D) blocks beta receptors and stimulates alpha receptors.
A

A

How well did you know this?
1
Not at all
2
3
4
5
Perfectly
11
Q
  1. A patient with orthopnea:
    A) experiences dyspnea during periods of exertion.
    B) prefers a semisitting position to facilitate breathing. C) experiences worsened dyspnea while lying down. D) sleeps in a recliner due to severe right heart failure.
A

C

How well did you know this?
1
Not at all
2
3
4
5
Perfectly
12
Q
  1. Myocardial ischemia occurs when the heart muscle:
    A) is deprived of oxygen because of a blocked coronary artery.
    B) undergoes necrosis because of prolonged oxygen deprivation.
    C) suffers oxygen deprivation secondary to coronary vasodilation. D) experiences a decreased oxygen demand and an increased supply.
A

A

How well did you know this?
1
Not at all
2
3
4
5
Perfectly
13
Q
  1. Infarctions of the inferior myocardial wall are MOST often caused by: A) blockage of the left coronary artery.
    B) acute spasm of the circumflex artery.
    C) occlusion of the right coronary artery.
    D) a blocked left anterior descending artery.
A

C

How well did you know this?
1
Not at all
2
3
4
5
Perfectly
14
Q
99. Which of the following patients would MOST likely present with atypical signs and symptoms of an acute myocardial infarction?
A) 49-year-old obese man
B) 58-year-old diabetic woman
C) 60-year-old man with anxiety
D) 71-year-old woman with hypertension
A

B

How well did you know this?
1
Not at all
2
3
4
5
Perfectly
15
Q
  1. What is the MOST appropriate sequence of treatment for a patient with a suspected acute myocardial infarction?
    A) Oxygen, aspirin, nitroglycerin, morphine
    B) Oxygen, nitroglycerin, aspirin, morphine
    C) Aspirin, nitroglycerin, oxygen, morphine D) Morphine, oxygen, aspirin, nitroglycerin
A

A

How well did you know this?
1
Not at all
2
3
4
5
Perfectly
16
Q
  1. In a patient with left heart failure and pulmonary edema:
    A) the right atrium and ventricle pump against lower pressures, resulting in the systemic pooling of venous blood.
    B) diffusely collapsed alveoli cause blood from the right side of the heart to bypass the alveoli and return to the left side of the heart.
    C) increased pressure in the left atrium and pulmonary veins forces serum out of the pulmonary capillaries and into the alveoli.
    D) an acute myocardial infarction or chronic hypertension causes the left ventricle to pump against decreased afterload, resulting in hypoperfusion.
A

C

How well did you know this?
1
Not at all
2
3
4
5
Perfectly
17
Q
123. Pericardial tamponade can be differentiated from a tension pneumothorax by the presence of:
A) jugular venous distention.
B) a narrowing pulse pressure.
C) clear and equal breath sounds.
D) alterations in the QRS amplitude.
A

C

How well did you know this?
1
Not at all
2
3
4
5
Perfectly
18
Q
  1. Hypertension is present when the blood pressure:
    A) increases by 20 mm Hg above a person’s normal blood pressure. B) is consistently greater than 140/90 mm Hg while at rest.
    C) is above 160 mm Hg systolic during strenuous exertion.
    D) rises acutely during an emotionally stressful situation.
A

B

How well did you know this?
1
Not at all
2
3
4
5
Perfectly
19
Q
  1. A decreased cardiac output secondary to a heart rate greater than 150 beats/min is caused by:
    A) myocardial stretching due to increased preload.
    B) decreases in stroke volume and ventricular filling.
    C) increased automaticity of the cardiac pacemaker. D) ectopic pacemaker sites in the atria or ventricles.
A

B

How well did you know this?
1
Not at all
2
3
4
5
Perfectly
20
Q
  1. A decreased cardiac output secondary to a heart rate greater than 150 beats/min is caused by:
    A) myocardial stretching due to increased preload.
    B) decreases in stroke volume and ventricular filling.
    C) increased automaticity of the cardiac pacemaker. D) ectopic pacemaker sites in the atria or ventricles.
A

B

How well did you know this?
1
Not at all
2
3
4
5
Perfectly
21
Q
  1. An electrical wave moving in the direction of a positive electrode will: A) cause a positive deflection on the ECG.
    B) produce a significant amount of artifact.
    C) cause a negative deflection on the ECG.
    D) manifest with narrow QRS complexes.
A

A

How well did you know this?
1
Not at all
2
3
4
5
Perfectly
22
Q
  1. Q waves are considered abnormal or pathologic if they are:
    A) greater than 0.02 seconds wide and consistently precede the R wave. B) more than one third the overall height of the QRS complex in lead II. C) not visible in leads I or II when the QRS gain sensitivity is increased. D) present in a patient who is experiencing chest pressure or discomfort.
A

B

How well did you know this?
1
Not at all
2
3
4
5
Perfectly
23
Q

A pathologic Q wave:
A) generally indicates that an acute myocardial infarction has occurred within the past hour. B) is deeper than one quarter of the height of the R wave and indicates injury.
C) is wider than 0.04 seconds and indicates that a myocardial infarction occurred in the past. D) can only be substantiated by viewing at least two previous 12-lead ECGs.

A

C

How well did you know this?
1
Not at all
2
3
4
5
Perfectly
24
Q
  1. The downslope of the T wave:
    A) is the point of ventricular repolarization to which a defibrillator is synchronized to deliver electrical energy.
    B) is the strongest part of ventricular depolarization and is often the origin of dangerous ventricular arrhythmias.
    C) represents a state of absolute ventricular refractoriness in which another impulse cannot cause depolarization.
    D) represents a vulnerable period during which a strong impulse could cause depolarization, resulting in a lethal arrhythmia.
A

D

How well did you know this?
1
Not at all
2
3
4
5
Perfectly
25
Q
  1. An ST segment that is more than 1 mm above the isoelectric line: A) indicates myocardial ischemia.
    B) is clinically insignificant in lead II.
    C) must be substantiated by a 12-lead ECG.
    D) is a definitive sign of myocardial injury.
A

C

How well did you know this?
1
Not at all
2
3
4
5
Perfectly
26
Q
  1. A right ventricular infarction is characterized by:
    A) ST-segment elevation greater than 1 mm in lead V5R and ST-segment depression in leads II,
    III, and aVF.
    B) ST-segment elevation greater than 1 mm in lead V4R and ST-segment elevation in leads II,
    III, and aVF.
    C) ST-segment depression greater than 2 mm in lead V4R and ST-segment elevation in leads II,
    III, and aVF.
    D) ST-segment elevation greater than 2 mm in lead V5R and ST-segment elevation in leads II,
    III, and aVF.
A

B

How well did you know this?
1
Not at all
2
3
4
5
Perfectly
27
Q
  1. A key to interpreting a Mobitz type II second-degree heart block is to remember that:
    A) unlike a Mobitz type I second-degree heart block, a type II heart block is always regular.
    B) in this type of heart block, the PR interval gets progressively longer until a P wave is not conducted.
    C) the PR interval of all of the conducted P waves and their corresponding QRS complexes is constant.
    D) most type II second degree AV blocks have more than two nonconducted P waves that occur in succession.
A

C

How well did you know this?
1
Not at all
2
3
4
5
Perfectly
28
Q
  1. Which of the following occurs at the AV node during a third-degree heart block? A) There is an abnormal delay in conducting impulses.
    B) Every third impulse is allowed to enter the ventricles.
    C) Impulses bypass the AV node and enter the ventricles.
    D) All impulses are blocked from entering the ventricles.
A

D

How well did you know this?
1
Not at all
2
3
4
5
Perfectly
29
Q
  1. Any electrical impulse that originates in the ventricles will produce: A) wide QRS complexes and a rate between 20 and 40 beats/min.
    B) a rapid rhythm with wide QRS complexes and no pulse.
    C) low-amplitude QRS complexes and dissociated P waves.
    D) bizarre-looking QRS complexes and a rate less than 60 beats/min.
A

A

How well did you know this?
1
Not at all
2
3
4
5
Perfectly
30
Q
  1. Premature ventricular complexes (PVCs) that originate from different sites in the ventricle: A) are called unifocal PVCs.
    B) produce a palpable pulse.
    C) are also called fusion PVCs.
    D) will appear differently on the ECG.
A

D

How well did you know this?
1
Not at all
2
3
4
5
Perfectly
31
Q
  1. Ventricular fibrillation occurs when:
    A) the ventricles quiver rather than contract normally, while organized atrial contractions continue as normal.
    B) the ventricles become the primary pacemaker for the heart, resulting in a rapid and irregular ventricular rhythm.
    C) many different cells in the heart depolarize independently rather than in response to an impulse from the SA node.
    D) cardiac cells in the ventricles fail to completely repolarize, resulting in a decrease in ventricular automaticity.
A

C

How well did you know this?
1
Not at all
2
3
4
5
Perfectly
32
Q
  1. The firing of an artificial ventricular pacemaker causes: A) a change in the shape of the preceding P waves.
    B) a vertical spike followed by a wide QRS complex.
    C) a small spike followed by a narrow QRS complex.
    D) a wide QRS complex followed by a vertical spike.
A

B

How well did you know this?
1
Not at all
2
3
4
5
Perfectly
33
Q
  1. A demand pacemaker:
    A) generates pacing impulses only when it senses that the heart’s natural pacemaker has fallen below a preset rate.
    B) sends out single electrical impulses when the patient’s inherent pacemaker rate exceeds 150 beats/min.
    C) is easily identified on a cardiac rhythm strip by noting the presence of pacer spikes before all of the QRS complexes.
    D) attaches to the atria and the ventricles and only generates an impulse if it senses that the patient is in ventricular fibrillation.
A

A

How well did you know this?
1
Not at all
2
3
4
5
Perfectly
34
Q
  1. Lead I views the ________ wall of the heart, while lead aVF views the _________ wall of the heart.
    A) lateral, inferior
    B) septal, anterior
    C) posterior, septal D) anterior, inferior
A

A

How well did you know this?
1
Not at all
2
3
4
5
Perfectly
35
Q

. The precordial leads do NOT view the __________ wall of the heart. A) septal
B) inferior
C) anterior
D) lateral

A

B

How well did you know this?
1
Not at all
2
3
4
5
Perfectly
36
Q
  1. When viewing leads V3 and V4, you are looking at the _________ wall of the _________.
    A) septal, heart.
    B) lateral, left ventricle. C) anterior, left ventricle. D) inferior, right ventricle.
A

C

How well did you know this?
1
Not at all
2
3
4
5
Perfectly
37
Q
  1. Which of the following leads provides the BEST view of the anterolateral wall of the left ventricle?
    A) V2 to V3
    B) V4 to V6 C) V4 to V5 D) V5 to V6
A

B

How well did you know this?
1
Not at all
2
3
4
5
Perfectly
38
Q
225. Leads V1 to V3 allow you to view the \_\_\_\_\_\_\_\_ wall of the left ventricle.
A) septal
B) lateral
C) anterior
D) anteroseptal
A

D

How well did you know this?
1
Not at all
2
3
4
5
Perfectly
39
Q
  1. Anatomically contiguous leads view:
    A) opposite walls of the heart.
    B) only the lateral wall of the heart
    . C) the same general area of the heart.
    D) only the anterior wall of the heart.
A

C

40
Q
  1. Injury to the inferior wall of the myocardium would present with: A) T-wave inversion in leads V1 through V4.
    B) ST-segment elevation in leads II, III, and aVF. C) pathologic Q waves in leads V4 and V5.
    D) ST-segment depression in leads V5, V6, and aVL.
A

B

41
Q
  1. Ischemia to the anterior wall of the myocardium would present with: A) T-wave inversion in leads V3 and V4.
    B) ST-segment depression in leads I and aVL. C) T-wave inversion in leads II, III, and aVF. D) ST-segment elevation in leads V3 and V4.
A

A

42
Q
  1. If the ECG leads are applied correctly, the PQRST configuration should be inverted in lead: A) I.
    B) II.
    C) aVR.
    D) aVL.
A

C

43
Q
  1. You receive a call to a residence for a 44-year-old man who is “ill.” The patient, who receives dialysis treatments three times a week, tells you that he has missed his last two treatments because he was not feeling well. As your partner takes the patient’s vital signs, you apply the ECG, which reveals a sinus rhythm with tall T waves. The 12-lead ECG reveals a sinus rhythm with inverted complexes in lead aVR. On the basis of your clinical findings, you should be MOST suspicious that the patient is:
    A) hypocalcemic.
    B) hypernatremic.
    C) hyperkalemic.
    D) having an acute myocardial infarction.
A

C

44
Q
  1. Most patients with an ST-elevation myocardial infarction: A) will develop Q waves.
    B) heal without treatment.
    C) experience cardiac arrest.
    D) present without chest pain.
A

A

45
Q
230. A diagnosis of acute myocardial infarction is made if ST-segment \_\_\_\_\_\_\_\_\_\_ of \_\_\_ mm or more is seen in \_\_\_ or more contiguous leads. 
A) elevation, 1, two
B) depression, 2, one
C) elevation, 2, one
D) depression, 1, two
A

A

46
Q
  1. Injury to the inferior wall of the myocardium would present with: A) T-wave inversion in leads V1 through V4.
    B) ST-segment elevation in leads II, III, and aVF. C) pathologic Q waves in leads V4 and V5.
    D) ST-segment depression in leads V5, V6, and aVL.
A

B

47
Q
  1. You are assessing the 12-lead tracing of a 40-year-old man with chest pain and note ST- segment elevation in leads II, III, and aVF. Lead V4R shows 2-mm ST-segment elevation. The
    patient’s blood pressure is 88/58 mm Hg, and his heart rate is 72 beats/min and regular. He denies any significant past medical history but is allergic to salicylates. After placing the patient on oxygen and starting an IV line of normal saline, you should:
    A) administer up to 325 mg of baby aspirin.
    B) give 2-mg increments of morphine sulfate. C) start a dopamine infusion at 2 μg/kg/min. D) give crystalloid boluses to increase preload.
A

D

48
Q
  1. Anatomically contiguous leads view: A) opposite walls of the heart.
    B) only the lateral wall of the heart.
    C) the same general area of the heart.
    D) only the anterior wall of the heart.
A

C

49
Q
  1. Which of the following statements is correct? A) Lead I is contiguous with lead II.
    B) Lead II is contiguous with leads V6 and aVL.
    C) Lead V6 is contiguous with leads V4 and V5. D) Lead III is contiguous with leads II and aVF.
A

D

50
Q

S - Septal

A

VI AND V2

51
Q

A - Anterior

A

V3 and V4

52
Q

L - Lateral

A

I, V5, V6, AVL

53
Q

I - Inferior

A

II, III, AVF

54
Q
  1. Ischemia to the anterior wall of the myocardium would present with:
    A) T-wave inversion in leads V3 and V4.
    B) ST-segment depression in leads I and aVL. C) T-wave inversion in leads II, III, and aVF. D) ST-segment elevation in leads V3 and V4.
A

A

55
Q
  1. A pathologic Q wave:
    A) generally indicates that an acute myocardial infarction has occurred within the past hour. B) is deeper than one quarter of the height of the R wave and indicates injury.
    C) is wider than 0.04 seconds and indicates that a myocardial infarction occurred in the past. D) can only be substantiated by viewing at least two previous 12-lead ECGs.
A

C

56
Q
  1. Q waves are considered abnormal or pathologic if they are:
    A) greater than 0.02 seconds wide and consistently precede the R wave.
    B) more than one third the overall height of the QRS complex in lead II.
    C) not visible in leads I or II when the QRS gain sensitivity is increased.
    D) present in a patient who is experiencing chest pressure or discomfort.
A

B

57
Q
  1. Q waves are considered abnormal or pathologic if they are:
    A) greater than 0.02 seconds wide and consistently precede the R wave.
    B) more than one third the overall height of the QRS complex in lead II.
    C) not visible in leads I or II when the QRS gain sensitivity is increased.
    D) present in a patient who is experiencing chest pressure or discomfort.
A

B

58
Q
  1. Patients who are experiencing an infarction of the right ventricle: A) should not be given IV fluid boluses.
    B) often require high doses of nitroglycerin.
    C) are usually hypertensive and tachycardic.
    D) may present with significant hypotension.
A

D

59
Q
  1. When performing CPR on an adult patient in cardiac arrest, it is important to: A) deliver at least 80 to 90 compressions per minute.
    B) limit interruptions in chest compressions to 20 seconds.
    C) deliver forceful ventilations between compressions.
    D) allow the chest to fully recoil between compressions.
A

D

60
Q
  1. When performing CPR on an adult patient in cardiac arrest, it is important to: A) deliver at least 80 to 90 compressions per minute.
    B) limit interruptions in chest compressions to 20 seconds.
    C) deliver forceful ventilations between compressions.
    D) allow the chest to fully recoil between compressions.
A

D

61
Q
242. The proper compression-to-ventilation ratio for two-rescuer adult CPR when an oropharyngeal airway is in place is:
A) 5:1.
B) 15:2.
C) 30:2.
D) asynchronous.
A

C

62
Q
  1. Which of the following actions should NOT occur while CPR is in progress? A) Advanced airway placement
    B) Cardiac rhythm assessment
    C) Assessment for a palpable pulse
    D) Establishment of vascular access
A

B

63
Q
259. The recommended first-line treatment for third-degree heart block associated with bradycardia and hemodynamic compromise is:
A) atropine sulfate.
B) a dopamine infusion.
C) an epinephrine infusion. 
D) transcutaneous pacing.
A

D

64
Q
  1. Once an advanced airway device has been inserted into a cardiac arrest patient: A) you should deliver one breath every 5 to 6 seconds.
    B) ventilations are delivered at a rate of 8 to 10 breaths/min.
    C) the compressor should pause so ventilations can be given.
    D) chest compressions should be increased to 120 per minute.
A

B

65
Q
  1. Freshly oxygenated blood is returned to the left atrium through the: A) pulmonary veins.
    B) pulmonary arteries.
    C) superior vena cava.
    D) inferior vena cava.
A

A

66
Q
  1. Automaticity is defined as the ability of the heart to:
    A) generate an electrical impulse from the same site every time.
    B) spontaneously conduct an electrical impulse between cardiac cells.
    C) generate its own electrical impulses without stimulation from nerves. D) increase or decrease its heart rate based on the body’s metabolic needs.
A

C

67
Q
  1. Dysfunction of the mitral valve may cause backflow of blood into the: A) right ventricle.
    B) right atrium.
    C) left atrium.
    D) left ventricle.
A

C

68
Q
  1. Which of the following mechanisms causes hypertension?
    A) Arteriosclerosis results in increased elasticity of the arteries, causing vasodilation and increased arteriolar capacity.
    B) Atherosclerotic plaque narrows one or more of the coronary arteries, resulting in increased cardiac perfusion.
    C) Increased afterload stimulates the Frank-Starling reflex, which raises the pressure behind the blood leaving the heart.
    D) Heart rate that is persistently above 80 beats/min causes an increase in cardiac output and a resultant increase in blood pressure.
A

C

69
Q
  1. According to the Frank-Starling mechanism:
    A) systemic venous pooling of blood results in a decrease in preload.
    B) an increase in systolic blood pressure causes a reflex bradycardia.
    C) coronary artery perfusion is directly proportional to cardiac output.
    D) the length of myocardial fibers determines force of cardiac contraction.
A

D

70
Q
  1. With regard to the heart, ejection fraction is defined as the: A) volume of blood that enters the lungs.
    B) volume of blood ejected from both atria.
    C) percentage of blood ejected from the heart.
    D) percentage of blood returned to the heart.
A

C

71
Q
20. If the left ventricle contains 80 mL of blood before a contraction and ejects 60 mL during the contraction, the ejection fraction is:
A) 60%.
B) 65%.
C) 70%. 
D) 75%.
A

D

72
Q
63. If the left ventricle fills with 85 mL of blood and ejects 60 mL during a contraction, the ejection fraction is approximately \_\_\_%.
A) 55
B) 60 
C) 65 
D) 70
A

D

73
Q
33. Thousands of fibrils that are distributed throughout the ventricles, which represent the end of the cardiac conduction system, are called the:
A) bundle branches.
B) internodal pathways.
C) Purkinje fibers. 
D) cardiac myocytes.
A

C

74
Q
7. The numerous connections among the arterioles of the various coronary arteries, which allow for the development of alternate routes of blood flow if a larger coronary artery begins to narrow, are called:
A) cardiac myocytes.
B) the coronary sinus.
C) collateral circulation.
D) coronary microcirculation.
A

C

75
Q
  1. Administering a drug that possesses a positive chronotropic effect will have a direct effect on:
    A) stroke volume.
    B) blood pressure.
    C) cardiac output. D) the heart rate.
A

D

76
Q
  1. A medication that possesses a positive chronotropic effect is one that: A) decreases heart rate.
    B) increases heart rate.
    C) decreases cardiac contractility.
    D) increases the conduction of electricity.
A

B

77
Q
  1. A medication that possesses a negative chronotropic effect will: A) cause a decrease in the heart rate.
    B) cause an increase in blood pressure.
    C) decrease myocardial contractile force.
    D) increase cardiac electrical conduction velocity.
A

A

78
Q
  1. Stimulation of the parasympathetic nervous system:
    A) completely blocks the AV node, preventing ventricular depolarization.
    B) causes a decrease in the production of epinephrine and norepinephrine.
    C) is characterized by a large P wave and a PR interval that is shorter than normal. D) slows SA nodal discharge and decreases conduction through the AV node.
A

F

79
Q
50. Stimulation of the parasympathetic nervous system causes all of the following effects, EXCEPT:
A) negative inotropy.
B) increased salivation.
C) dilation of the pupils. 
D) negative chronotropy.
A

C

80
Q
  1. A physiologic effect of sympathetic nervous stimulation includes: A) dilation of the bronchioles.
    B) decreased conduction velocity.
    C) a negative dromotropic effect.
    D) dilation of the blood vessels.
A

A

81
Q
  1. Vasoconstriction occurs following stimulation of: A) beta-1 receptors.
    B) beta-2 receptors.
    C) alpha receptors.
    D) alpha and beta receptors.
A

C

82
Q
  1. The MOST effective drug for the treatment of non-vagal-induced bradycardia is: A) atropine.
    B) dopamine.
    C) epinephrine.
    D) metoprolol.
A

C

83
Q
  1. In the context of cardiac compromise, syncope occurs due to: A) an increase in vagal tone.
    B) a drop in cerebral perfusion.
    C) a sudden cardiac dysrhythmia. D) an acute increase in heart rate.
A

B

84
Q

Pain associated with an AMI is not influencing factors
98. The pain associated with an acute myocardial infarction:
A) radiates to the left or right arm in the majority of cases.
B) is not influenced by deep breathing or body movement.
C) is most often described as a sharp sensation in the chest.
D) is often relieved by two or three doses of sublingual nitroglycerin.

A

B

85
Q
  1. Which of the following statements regarding oxygen administration for a patient experiencing an acute myocardial infarction is correct?
    A) Evidence has shown that high (greater than 90%) concentrations of oxygen reduce mortality. B) In order to prevent hypoxic injury, do not give any patient with an acute myocardial infarction more than 2 L/min of oxygen.
    C) Treatment with oxygen should be individualized and titrated to maintain the SpO2 level above
    94%.
    D) Any patient experiencing an acute myocardial infarction should receive high-flow oxygen.
A

C

86
Q
  1. The MOST significant risk associated with the use of fibrinolytic therapy is: A) reocclusion.
    B) coagulation.
    C) anaphylaxis.
    D) hemorrhage.
A

S

87
Q
  1. Hypertension is present when the blood pressure:
    A) increases by 20 mm Hg above a person’s normal blood pressure. B) is consistently greater than 140/90 mm Hg while at rest.
    C) is above 160 mm Hg systolic during strenuous exertion.
    D) rises acutely during an emotionally stressful situation.
A

B

88
Q
  1. Which of the following mechanisms causes hypertension?
    A) Arteriosclerosis results in increased elasticity of the arteries, causing vasodilation and increased arteriolar capacity.
    B) Atherosclerotic plaque narrows one or more of the coronary arteries, resulting in increased cardiac perfusion.
    C) Increased afterload stimulates the Frank-Starling reflex, which raises the pressure behind the blood leaving the heart.
    D) Heart rate that is persistently above 80 beats/min causes an increase in cardiac output and a resultant increase in blood pressure.
A

C

89
Q
  1. The normal P wave duration is less than ___ milliseconds and the amplitude is less than ___ millimeters tall.
    A) 110; 2.5
    B) 120; 3.0
    C) 130; 3.5 D) 140; 4.0
A

A

90
Q
  1. Which of the following differentiates an atrial rhythm from a sinus rhythm? A) Tachycardia
    B) Profound bradycardia
    C) Dissociated P waves
    D) Varying shapes in P waves
A

D

91
Q

What is ventricular bigemeny
207. Ventricular bigeminy occurs when:
A) two premature ventricular complexes (PVCs) occur in a row. B) every second complex is a PVC.
C) at least two differently shaped PVCs occur.
D) a 6-second strip contains at least two PVCs.

A

B

92
Q
  1. When assessing an anxious patient who presents with tachycardia, you must: A) obtain a 12-lead ECG tracing before initiating any treatment.
    B) determine if the tachycardia is causing hemodynamic instability.
    C) prepare for cardioversion if the rate is less than 150 beats/min.
    D) administer diazepam or midazolam to facilitate your assessment.
A

B

93
Q
  1. Which of the following factors would present the GREATEST difficulty when distinguishing supraventricular tachycardia from ventricular tachycardia?
    A) Aberrant conduction
    B) Absence of P waves
    C) Retrograde conduction D) The rate of the rhythm
A

A

94
Q
  1. Which of the following statements regarding the SA node is correct? A) The SA node is the dominant cardiac pacemaker in healthy patients.
    B) SA nodal ischemia occurs when the left coronary artery is occluded.
    C) The SA node is located in the superior aspect of the right ventricle.
    D) Impulses generated by the SA node travel through the right atrium only.
A

A

95
Q

Why do we give epi in cardiac arrest? What does it increase?

A

Bcuz what?

96
Q
  1. Which of the following statements regarding the SA node is correct? A) The SA node is the dominant cardiac pacemaker in healthy patients.
    B) SA nodal ischemia occurs when the left coronary artery is occluded.
    C) The SA node is located in the superior aspect of the right ventricle.
    D) Impulses generated by the SA node travel through the right atrium only.
A

A

97
Q
  1. TheSAnode:
    A) cannot depolarize faster than 100 times/min.
    B) will outpace any slower conduction tissue.
    C) functions as the heart’s secondary pacemaker.
    D) has an intrinsic firing rate of 40 to 60 times per minute.
A

B